Prove $x equiv a pmod{p}$ and $x equiv a pmod{q}$ then $x equiv apmod{pq}$











up vote
3
down vote

favorite












$p$ and $q$ are distinct primes.



Where can I start with this proof?
It looks similar to the Chinese Remainder Theorem, but that deals with two different a values.










share|cite|improve this question
























  • but that deals with two different $a$ values - What makes you think that?
    – anon
    Sep 3 '12 at 15:59










  • Because for the explanations of Chinese Remainder Theorem I've read, they use something like a = x (mod p) and a = y (mod p)
    – Takkun
    Sep 3 '12 at 16:03






  • 2




    A couple of variables denoted with two different letters may take on two different values, but not necessarily unless they are explicitly stated to have distinct values.
    – anon
    Sep 3 '12 at 16:04

















up vote
3
down vote

favorite












$p$ and $q$ are distinct primes.



Where can I start with this proof?
It looks similar to the Chinese Remainder Theorem, but that deals with two different a values.










share|cite|improve this question
























  • but that deals with two different $a$ values - What makes you think that?
    – anon
    Sep 3 '12 at 15:59










  • Because for the explanations of Chinese Remainder Theorem I've read, they use something like a = x (mod p) and a = y (mod p)
    – Takkun
    Sep 3 '12 at 16:03






  • 2




    A couple of variables denoted with two different letters may take on two different values, but not necessarily unless they are explicitly stated to have distinct values.
    – anon
    Sep 3 '12 at 16:04















up vote
3
down vote

favorite









up vote
3
down vote

favorite











$p$ and $q$ are distinct primes.



Where can I start with this proof?
It looks similar to the Chinese Remainder Theorem, but that deals with two different a values.










share|cite|improve this question















$p$ and $q$ are distinct primes.



Where can I start with this proof?
It looks similar to the Chinese Remainder Theorem, but that deals with two different a values.







number-theory






share|cite|improve this question















share|cite|improve this question













share|cite|improve this question




share|cite|improve this question








edited Sep 3 '12 at 16:08









Thomas Russell

7,79632550




7,79632550










asked Sep 3 '12 at 15:57









Takkun

263311




263311












  • but that deals with two different $a$ values - What makes you think that?
    – anon
    Sep 3 '12 at 15:59










  • Because for the explanations of Chinese Remainder Theorem I've read, they use something like a = x (mod p) and a = y (mod p)
    – Takkun
    Sep 3 '12 at 16:03






  • 2




    A couple of variables denoted with two different letters may take on two different values, but not necessarily unless they are explicitly stated to have distinct values.
    – anon
    Sep 3 '12 at 16:04




















  • but that deals with two different $a$ values - What makes you think that?
    – anon
    Sep 3 '12 at 15:59










  • Because for the explanations of Chinese Remainder Theorem I've read, they use something like a = x (mod p) and a = y (mod p)
    – Takkun
    Sep 3 '12 at 16:03






  • 2




    A couple of variables denoted with two different letters may take on two different values, but not necessarily unless they are explicitly stated to have distinct values.
    – anon
    Sep 3 '12 at 16:04


















but that deals with two different $a$ values - What makes you think that?
– anon
Sep 3 '12 at 15:59




but that deals with two different $a$ values - What makes you think that?
– anon
Sep 3 '12 at 15:59












Because for the explanations of Chinese Remainder Theorem I've read, they use something like a = x (mod p) and a = y (mod p)
– Takkun
Sep 3 '12 at 16:03




Because for the explanations of Chinese Remainder Theorem I've read, they use something like a = x (mod p) and a = y (mod p)
– Takkun
Sep 3 '12 at 16:03




2




2




A couple of variables denoted with two different letters may take on two different values, but not necessarily unless they are explicitly stated to have distinct values.
– anon
Sep 3 '12 at 16:04






A couple of variables denoted with two different letters may take on two different values, but not necessarily unless they are explicitly stated to have distinct values.
– anon
Sep 3 '12 at 16:04












3 Answers
3






active

oldest

votes

















up vote
6
down vote



accepted










Hint $ $ Below are few proofs of this constant-case CCRT of Chinese Remainder Theorem (CRT). The first three proofs work for for arbitrary coprime naturals $rm,p,q.$



$rm(1) x equiv apmod {pq}:$ is clearly a solution, and the solution is $color{#C00}{unique}$ mod $rm,pq,$ by CRT.



$rm(2) p,q:|:x!-!aiff lcm(p,q):|:x!-!a.:$ Further $rm:(p,q)=1iff:lcm(p,q) = pq.$



$(3) , $ By Euclid's Lemma: $rm:(p,q)=1, p:|:qn =:x!-!a:Rightarrow:p:|:n:Rightarrow:pq:|:nq = x!-!a.$



$rm(4) , $ Since the prime factorization of $rm,x!-!a,$ is $color{#C00}{unique}$, and the prime $rm,p,$ occurs in one factorization, and the distinct prime $rm,q,$ occurs in another, both factorizations are the same up to order, hence contain both $rm,p,$ and $rm,q,:$ hence have $rm,pq,$ as a divisor.



Remark $ $ This constant-case optimization of CRT arises frequently in practice so is well-worth memorizing, e.g. see some prior posts for many examples.



Quite frequently, $color{#C00}{uniqueness} theorems$ provide powerful tools for proving equalities.






share|cite|improve this answer























  • in (2) do you mean to say $gcd(p, q) = 1 iff lcm(p, q) = pq$
    – CodeKingPlusPlus
    Sep 23 '12 at 23:17










  • @Code Yes, $rm:(x,y):$ means $rm:gcd(x,y):$ in number theory (common notation).
    – Bill Dubuque
    Sep 23 '12 at 23:37


















up vote
4
down vote













Let $[A,B]=lcm(A,B)$ and $(A,B)=gcd(A,B)$



If $p,q$ are different integers, $pmid(x-a)$ and $qmid(x-a)implies [p,q]mid(x-a)$



We know $[p,q]cdot (p,q)=pcdot q$



If $(p,q)=1, [p,q]=pcdot q$



If $p,q$ are distinct primes, $(p,q)=1$






share|cite|improve this answer






























    up vote
    3
    down vote













    Let $y=x-a$. We want to show that if $p$ divides $y$ and $q$ divides $y$ then $pq$ divides $y$.



    Since $p$ divides $y$, we have $y=pz$ for some $z$. Thus $q$ divides $pz$. Since $q$ is prime, this implies $q$ divides $p$ or $q$ divides $z$. But $q$ cannot divide $p$, so $q$ divides $z$. Suppose that $z=qw$. Then $y=pqw$.






    share|cite|improve this answer























      Your Answer





      StackExchange.ifUsing("editor", function () {
      return StackExchange.using("mathjaxEditing", function () {
      StackExchange.MarkdownEditor.creationCallbacks.add(function (editor, postfix) {
      StackExchange.mathjaxEditing.prepareWmdForMathJax(editor, postfix, [["$", "$"], ["\\(","\\)"]]);
      });
      });
      }, "mathjax-editing");

      StackExchange.ready(function() {
      var channelOptions = {
      tags: "".split(" "),
      id: "69"
      };
      initTagRenderer("".split(" "), "".split(" "), channelOptions);

      StackExchange.using("externalEditor", function() {
      // Have to fire editor after snippets, if snippets enabled
      if (StackExchange.settings.snippets.snippetsEnabled) {
      StackExchange.using("snippets", function() {
      createEditor();
      });
      }
      else {
      createEditor();
      }
      });

      function createEditor() {
      StackExchange.prepareEditor({
      heartbeatType: 'answer',
      convertImagesToLinks: true,
      noModals: true,
      showLowRepImageUploadWarning: true,
      reputationToPostImages: 10,
      bindNavPrevention: true,
      postfix: "",
      imageUploader: {
      brandingHtml: "Powered by u003ca class="icon-imgur-white" href="https://imgur.com/"u003eu003c/au003e",
      contentPolicyHtml: "User contributions licensed under u003ca href="https://creativecommons.org/licenses/by-sa/3.0/"u003ecc by-sa 3.0 with attribution requiredu003c/au003e u003ca href="https://stackoverflow.com/legal/content-policy"u003e(content policy)u003c/au003e",
      allowUrls: true
      },
      noCode: true, onDemand: true,
      discardSelector: ".discard-answer"
      ,immediatelyShowMarkdownHelp:true
      });


      }
      });














      draft saved

      draft discarded


















      StackExchange.ready(
      function () {
      StackExchange.openid.initPostLogin('.new-post-login', 'https%3a%2f%2fmath.stackexchange.com%2fquestions%2f190514%2fprove-x-equiv-a-pmodp-and-x-equiv-a-pmodq-then-x-equiv-a-pmodpq%23new-answer', 'question_page');
      }
      );

      Post as a guest















      Required, but never shown

























      3 Answers
      3






      active

      oldest

      votes








      3 Answers
      3






      active

      oldest

      votes









      active

      oldest

      votes






      active

      oldest

      votes








      up vote
      6
      down vote



      accepted










      Hint $ $ Below are few proofs of this constant-case CCRT of Chinese Remainder Theorem (CRT). The first three proofs work for for arbitrary coprime naturals $rm,p,q.$



      $rm(1) x equiv apmod {pq}:$ is clearly a solution, and the solution is $color{#C00}{unique}$ mod $rm,pq,$ by CRT.



      $rm(2) p,q:|:x!-!aiff lcm(p,q):|:x!-!a.:$ Further $rm:(p,q)=1iff:lcm(p,q) = pq.$



      $(3) , $ By Euclid's Lemma: $rm:(p,q)=1, p:|:qn =:x!-!a:Rightarrow:p:|:n:Rightarrow:pq:|:nq = x!-!a.$



      $rm(4) , $ Since the prime factorization of $rm,x!-!a,$ is $color{#C00}{unique}$, and the prime $rm,p,$ occurs in one factorization, and the distinct prime $rm,q,$ occurs in another, both factorizations are the same up to order, hence contain both $rm,p,$ and $rm,q,:$ hence have $rm,pq,$ as a divisor.



      Remark $ $ This constant-case optimization of CRT arises frequently in practice so is well-worth memorizing, e.g. see some prior posts for many examples.



      Quite frequently, $color{#C00}{uniqueness} theorems$ provide powerful tools for proving equalities.






      share|cite|improve this answer























      • in (2) do you mean to say $gcd(p, q) = 1 iff lcm(p, q) = pq$
        – CodeKingPlusPlus
        Sep 23 '12 at 23:17










      • @Code Yes, $rm:(x,y):$ means $rm:gcd(x,y):$ in number theory (common notation).
        – Bill Dubuque
        Sep 23 '12 at 23:37















      up vote
      6
      down vote



      accepted










      Hint $ $ Below are few proofs of this constant-case CCRT of Chinese Remainder Theorem (CRT). The first three proofs work for for arbitrary coprime naturals $rm,p,q.$



      $rm(1) x equiv apmod {pq}:$ is clearly a solution, and the solution is $color{#C00}{unique}$ mod $rm,pq,$ by CRT.



      $rm(2) p,q:|:x!-!aiff lcm(p,q):|:x!-!a.:$ Further $rm:(p,q)=1iff:lcm(p,q) = pq.$



      $(3) , $ By Euclid's Lemma: $rm:(p,q)=1, p:|:qn =:x!-!a:Rightarrow:p:|:n:Rightarrow:pq:|:nq = x!-!a.$



      $rm(4) , $ Since the prime factorization of $rm,x!-!a,$ is $color{#C00}{unique}$, and the prime $rm,p,$ occurs in one factorization, and the distinct prime $rm,q,$ occurs in another, both factorizations are the same up to order, hence contain both $rm,p,$ and $rm,q,:$ hence have $rm,pq,$ as a divisor.



      Remark $ $ This constant-case optimization of CRT arises frequently in practice so is well-worth memorizing, e.g. see some prior posts for many examples.



      Quite frequently, $color{#C00}{uniqueness} theorems$ provide powerful tools for proving equalities.






      share|cite|improve this answer























      • in (2) do you mean to say $gcd(p, q) = 1 iff lcm(p, q) = pq$
        – CodeKingPlusPlus
        Sep 23 '12 at 23:17










      • @Code Yes, $rm:(x,y):$ means $rm:gcd(x,y):$ in number theory (common notation).
        – Bill Dubuque
        Sep 23 '12 at 23:37













      up vote
      6
      down vote



      accepted







      up vote
      6
      down vote



      accepted






      Hint $ $ Below are few proofs of this constant-case CCRT of Chinese Remainder Theorem (CRT). The first three proofs work for for arbitrary coprime naturals $rm,p,q.$



      $rm(1) x equiv apmod {pq}:$ is clearly a solution, and the solution is $color{#C00}{unique}$ mod $rm,pq,$ by CRT.



      $rm(2) p,q:|:x!-!aiff lcm(p,q):|:x!-!a.:$ Further $rm:(p,q)=1iff:lcm(p,q) = pq.$



      $(3) , $ By Euclid's Lemma: $rm:(p,q)=1, p:|:qn =:x!-!a:Rightarrow:p:|:n:Rightarrow:pq:|:nq = x!-!a.$



      $rm(4) , $ Since the prime factorization of $rm,x!-!a,$ is $color{#C00}{unique}$, and the prime $rm,p,$ occurs in one factorization, and the distinct prime $rm,q,$ occurs in another, both factorizations are the same up to order, hence contain both $rm,p,$ and $rm,q,:$ hence have $rm,pq,$ as a divisor.



      Remark $ $ This constant-case optimization of CRT arises frequently in practice so is well-worth memorizing, e.g. see some prior posts for many examples.



      Quite frequently, $color{#C00}{uniqueness} theorems$ provide powerful tools for proving equalities.






      share|cite|improve this answer














      Hint $ $ Below are few proofs of this constant-case CCRT of Chinese Remainder Theorem (CRT). The first three proofs work for for arbitrary coprime naturals $rm,p,q.$



      $rm(1) x equiv apmod {pq}:$ is clearly a solution, and the solution is $color{#C00}{unique}$ mod $rm,pq,$ by CRT.



      $rm(2) p,q:|:x!-!aiff lcm(p,q):|:x!-!a.:$ Further $rm:(p,q)=1iff:lcm(p,q) = pq.$



      $(3) , $ By Euclid's Lemma: $rm:(p,q)=1, p:|:qn =:x!-!a:Rightarrow:p:|:n:Rightarrow:pq:|:nq = x!-!a.$



      $rm(4) , $ Since the prime factorization of $rm,x!-!a,$ is $color{#C00}{unique}$, and the prime $rm,p,$ occurs in one factorization, and the distinct prime $rm,q,$ occurs in another, both factorizations are the same up to order, hence contain both $rm,p,$ and $rm,q,:$ hence have $rm,pq,$ as a divisor.



      Remark $ $ This constant-case optimization of CRT arises frequently in practice so is well-worth memorizing, e.g. see some prior posts for many examples.



      Quite frequently, $color{#C00}{uniqueness} theorems$ provide powerful tools for proving equalities.







      share|cite|improve this answer














      share|cite|improve this answer



      share|cite|improve this answer








      edited Nov 19 at 18:12

























      answered Sep 3 '12 at 16:18









      Bill Dubuque

      207k29189624




      207k29189624












      • in (2) do you mean to say $gcd(p, q) = 1 iff lcm(p, q) = pq$
        – CodeKingPlusPlus
        Sep 23 '12 at 23:17










      • @Code Yes, $rm:(x,y):$ means $rm:gcd(x,y):$ in number theory (common notation).
        – Bill Dubuque
        Sep 23 '12 at 23:37


















      • in (2) do you mean to say $gcd(p, q) = 1 iff lcm(p, q) = pq$
        – CodeKingPlusPlus
        Sep 23 '12 at 23:17










      • @Code Yes, $rm:(x,y):$ means $rm:gcd(x,y):$ in number theory (common notation).
        – Bill Dubuque
        Sep 23 '12 at 23:37
















      in (2) do you mean to say $gcd(p, q) = 1 iff lcm(p, q) = pq$
      – CodeKingPlusPlus
      Sep 23 '12 at 23:17




      in (2) do you mean to say $gcd(p, q) = 1 iff lcm(p, q) = pq$
      – CodeKingPlusPlus
      Sep 23 '12 at 23:17












      @Code Yes, $rm:(x,y):$ means $rm:gcd(x,y):$ in number theory (common notation).
      – Bill Dubuque
      Sep 23 '12 at 23:37




      @Code Yes, $rm:(x,y):$ means $rm:gcd(x,y):$ in number theory (common notation).
      – Bill Dubuque
      Sep 23 '12 at 23:37










      up vote
      4
      down vote













      Let $[A,B]=lcm(A,B)$ and $(A,B)=gcd(A,B)$



      If $p,q$ are different integers, $pmid(x-a)$ and $qmid(x-a)implies [p,q]mid(x-a)$



      We know $[p,q]cdot (p,q)=pcdot q$



      If $(p,q)=1, [p,q]=pcdot q$



      If $p,q$ are distinct primes, $(p,q)=1$






      share|cite|improve this answer



























        up vote
        4
        down vote













        Let $[A,B]=lcm(A,B)$ and $(A,B)=gcd(A,B)$



        If $p,q$ are different integers, $pmid(x-a)$ and $qmid(x-a)implies [p,q]mid(x-a)$



        We know $[p,q]cdot (p,q)=pcdot q$



        If $(p,q)=1, [p,q]=pcdot q$



        If $p,q$ are distinct primes, $(p,q)=1$






        share|cite|improve this answer

























          up vote
          4
          down vote










          up vote
          4
          down vote









          Let $[A,B]=lcm(A,B)$ and $(A,B)=gcd(A,B)$



          If $p,q$ are different integers, $pmid(x-a)$ and $qmid(x-a)implies [p,q]mid(x-a)$



          We know $[p,q]cdot (p,q)=pcdot q$



          If $(p,q)=1, [p,q]=pcdot q$



          If $p,q$ are distinct primes, $(p,q)=1$






          share|cite|improve this answer














          Let $[A,B]=lcm(A,B)$ and $(A,B)=gcd(A,B)$



          If $p,q$ are different integers, $pmid(x-a)$ and $qmid(x-a)implies [p,q]mid(x-a)$



          We know $[p,q]cdot (p,q)=pcdot q$



          If $(p,q)=1, [p,q]=pcdot q$



          If $p,q$ are distinct primes, $(p,q)=1$







          share|cite|improve this answer














          share|cite|improve this answer



          share|cite|improve this answer








          edited Sep 3 '12 at 16:06

























          answered Sep 3 '12 at 16:01









          lab bhattacharjee

          221k15155273




          221k15155273






















              up vote
              3
              down vote













              Let $y=x-a$. We want to show that if $p$ divides $y$ and $q$ divides $y$ then $pq$ divides $y$.



              Since $p$ divides $y$, we have $y=pz$ for some $z$. Thus $q$ divides $pz$. Since $q$ is prime, this implies $q$ divides $p$ or $q$ divides $z$. But $q$ cannot divide $p$, so $q$ divides $z$. Suppose that $z=qw$. Then $y=pqw$.






              share|cite|improve this answer



























                up vote
                3
                down vote













                Let $y=x-a$. We want to show that if $p$ divides $y$ and $q$ divides $y$ then $pq$ divides $y$.



                Since $p$ divides $y$, we have $y=pz$ for some $z$. Thus $q$ divides $pz$. Since $q$ is prime, this implies $q$ divides $p$ or $q$ divides $z$. But $q$ cannot divide $p$, so $q$ divides $z$. Suppose that $z=qw$. Then $y=pqw$.






                share|cite|improve this answer

























                  up vote
                  3
                  down vote










                  up vote
                  3
                  down vote









                  Let $y=x-a$. We want to show that if $p$ divides $y$ and $q$ divides $y$ then $pq$ divides $y$.



                  Since $p$ divides $y$, we have $y=pz$ for some $z$. Thus $q$ divides $pz$. Since $q$ is prime, this implies $q$ divides $p$ or $q$ divides $z$. But $q$ cannot divide $p$, so $q$ divides $z$. Suppose that $z=qw$. Then $y=pqw$.






                  share|cite|improve this answer














                  Let $y=x-a$. We want to show that if $p$ divides $y$ and $q$ divides $y$ then $pq$ divides $y$.



                  Since $p$ divides $y$, we have $y=pz$ for some $z$. Thus $q$ divides $pz$. Since $q$ is prime, this implies $q$ divides $p$ or $q$ divides $z$. But $q$ cannot divide $p$, so $q$ divides $z$. Suppose that $z=qw$. Then $y=pqw$.







                  share|cite|improve this answer














                  share|cite|improve this answer



                  share|cite|improve this answer








                  edited Sep 3 '12 at 16:30

























                  answered Sep 3 '12 at 16:08









                  André Nicolas

                  451k36421805




                  451k36421805






























                      draft saved

                      draft discarded




















































                      Thanks for contributing an answer to Mathematics Stack Exchange!


                      • Please be sure to answer the question. Provide details and share your research!

                      But avoid



                      • Asking for help, clarification, or responding to other answers.

                      • Making statements based on opinion; back them up with references or personal experience.


                      Use MathJax to format equations. MathJax reference.


                      To learn more, see our tips on writing great answers.





                      Some of your past answers have not been well-received, and you're in danger of being blocked from answering.


                      Please pay close attention to the following guidance:


                      • Please be sure to answer the question. Provide details and share your research!

                      But avoid



                      • Asking for help, clarification, or responding to other answers.

                      • Making statements based on opinion; back them up with references or personal experience.


                      To learn more, see our tips on writing great answers.




                      draft saved


                      draft discarded














                      StackExchange.ready(
                      function () {
                      StackExchange.openid.initPostLogin('.new-post-login', 'https%3a%2f%2fmath.stackexchange.com%2fquestions%2f190514%2fprove-x-equiv-a-pmodp-and-x-equiv-a-pmodq-then-x-equiv-a-pmodpq%23new-answer', 'question_page');
                      }
                      );

                      Post as a guest















                      Required, but never shown





















































                      Required, but never shown














                      Required, but never shown












                      Required, but never shown







                      Required, but never shown

































                      Required, but never shown














                      Required, but never shown












                      Required, but never shown







                      Required, but never shown







                      Popular posts from this blog

                      Plaza Victoria

                      In PowerPoint, is there a keyboard shortcut for bulleted / numbered list?

                      How to put 3 figures in Latex with 2 figures side by side and 1 below these side by side images but in...